Talk:2022 AMC 10A Problems/Problem 20

< Talk:2022 AMC 10A Problems
Revision as of 21:23, 6 October 2023 by Pixelpyguy (talk | contribs) (Created page with "For Solution 1: What is the contradiction with <math>b=28</math>?")
(diff) ← Older revision | Latest revision (diff) | Newer revision → (diff)

For Solution 1:

What is the contradiction with $b=28$?